Neuro Q's from druma

Réussis tes devoirs et examens dès maintenant avec Quizwiz!

A 19-year-old college student is brought to the ED after having a generalized tonic-clonic seizure. His roommate says that he has been sick for 2-3 days and describes a febrile illness, headache, and irritability. The patient is somnolent and poorly responsive in the ED. There are no focal neurological deficits on exam. Which of the following is the most likely diagnosis? A) Viral encephalitis B) Bacterial meningitis C) Subarachnoid hemorrhage D) Bacterial abscess E) Viral meningitis

A. Viral encephalitis Explanation Herpes simplex virus (HSV) is the most likely etiology. This is the most common cause of sporadic viral encephalitis, 10 - 20% of all annual cases of viral encephalitis. (On the Boards, be on the lookout for West Nile virus as the most common cause of "epidemic" viral encephalitis.) In adults, viral encephalitis due to HSV is due to HSV-1 in more than 90% of cases (in neonates, 80-90% are due to HSV-2). Primary HSV-1 infection accounts for 25% of adult cases. It causes fever, headache, and a rapidly worsening altered mental status (often including personality changes). CSF may show lymphocyte pleocytosis and increased CSF protein, 84% have increased RBCs in CSF. Neuroimaging (usually MRI) may show hemorrhagic destruction of the temporal lobes; however, these findings may not be present for several days. EEG may show temporal lobe abnormalities such as periodic lateralized epileptiform discharges (PLEDs). Polymerase chain reaction (PCR) for HSV is both specific (94 - 100%) and sensitive (98%) and is often used to confirm the diagnosis. Viral cultures are rarely positive. The treatment is acyclovir, 10 mg/kg every 8 hours, usually for a three-week course. The diagnosis is much less likely to be a bacterial abscess. Meningitis, by itself, does not cause personality changes or this degree of encephalopathy. Although a subarachnoid hemorrhage can cause neck stiffness and somnolence, the febrile illness and history of irritability is more suggestive of an encephalitis.

A 30-year-old man has a sudden severe headache. He thinks that when the headache started, that he may have "passed out" for a "second." Upon arrival in the emergency department, he has neck stiffness, photophobia, and a blood pressure of 170/96. His exam is normal otherwise. CT scan shows blood in the suprasellar cistern and left sylvian fissure. What is the next most appropriate test? A) 4-vessel angiography. B) Lumbar puncture. C) MRI brain. D) Carotid Doppler. E) Call neurosurgery!

Answer A. 4-vessel angiography. Explanation Because we can see the blood on CT, we know he has had a subarachnoid hemorrhage, so the LP is unlikely to add to what we already know. Since aneurismal rupture is highly suspect, and can be multiple in 10-25%, 4-vessel angiography is needed both to find the cause of the bleed as well as identifying other aneurysms, if present. MRI may also show aneurysms, but has a lower resolution for aneurysms below a certain size (usually below 3-4 mm), whereas angiography will show these. Calling neurosurgery is a good idea, but they will likely ask for the angiography results! As for Doppler—these are helpful in identifying the cause of an ischemic stroke, but are unlikely to help in the evaluation of subarachnoid hemorrhage.

You are asked to evaluate a patient who was admitted with complaints consistent with acute transverse myelitis. The patient has developed symptoms over the preceding 48 hours. Past history is remarkable for a one-day history 3 years ago of an acute vision disturbance. A diagnosis of optic neuritis was made at that time. Workup during that episode failed to establish an additional diagnosis. Which of the following factors would be associated with the most severe prognosis? A) Age of onset older than 50 years B) Optic neuritis as initial symptom C) Female gender D) Relapsing-remitting cycles

Answer A. Age of onset older than 50 years Explanation Multiple sclerosis (MS) requires demyelinating processes in two separate CNS locations, at least 4 weeks between episodes, and no other diagnosis that can account for the symptoms. Unless evidence of older lesions can be demonstrated when the patient initially presents, a diagnosis of MS cannot be established. If additional workup does not reveal an alternate diagnosis, these cases are referred to as a clinically isolated syndrome (CIS). The prognosis for patients with MS is best for younger females with a remitting-relapsing disease course. Optic neuritis as an initial symptom is also a favorable prognostic indicator. Of the choices provided, a patient older than 50 years would be expected to have the most severe clinical course.

A 50-year-old man comes to the office with "clumsiness." It has been getting worse gradually over the past few weeks. At first, it involved the right foot. Later, he noticed problems in both feet and was "tripping over" his own feet. In the past week, he has noticed a change in his voice: It now sounds more "nasal." He denies recent infections. He has had no fever, chills, nausea, or vomiting. On examination, he has a "nasal" sounding voice, but his speech is fluent. Naming and repetition are excellent. He has weakness of the legs more so than the arms. He has fasciculations of several muscles and is hyperreflexic in all 4 extremities. What is the most likely diagnosis? A) Amyotrophic lateral sclerosis (ALS or Lou Gehrig disease) B) Herpes encephalitis C) Brain tumor in the dominant hemisphere D) Huntington disease

Answer A. Amyotrophic lateral sclerosis (ALS or Lou Gehrig disease) Explanation Amyotrophic lateral sclerosis affects people in their 40s to 60s. It usually causes weakness or clumsiness of the limbs first. (In 75%, the limbs are affected first.) When it begins, it is usually asymmetric, but later it will affect both sides. Later, ALS causes "bulbar muscle" dysfunction. These are the muscles that help to coordinate speech and swallowing; in our patient in question, this was the cause of the "nasal" voice. Characteristic of ALS is the combination of upper motor neuron signs (the hyperreflexia) and lower motor neuron signs (the fasciculations). In fact, it is this combination of findings that should make you think of ALS. Huntington's also affects people in their 40s, but usually causes dystonia or abnormal movements of the limbs; e.g., choreiform, which are jerk-like or athetoid, which are writhing movements. Huntington's also often causes anxiety, depression, or mood changes. One similarity between ALS and Huntington's is that both can progress quickly. However, Huntington's would not cause hyperreflexia. A brain tumor would most likely cause gradually worsening focal neurological deficits such as weakness, numbness, and hyperreflexia. However, the signs would be contralateral to the hemisphere in which the tumor was enlarging and would not be bilateral. A tumor in the dominant hemisphere could affect language, but would likely produce an aphasia which would manifest as trouble with naming, repetition, or production of speech. (He has none of these findings.) The one instance in which a tumor could affect both sides of the body as well as speech would be when the tumor is located in the brainstem. In this instance, the patient would likely have one or more cranial nerve deficits. Herpes encephalitis most often presents with fever, personality changes, etc.

A 67-year-old man is brought to the ED with a severe headache that he describes as the worst he has ever experienced. He has been feeling unwell for the past day and has a fever of 101.5° F. On exam, he is mildly sleepy and has neck stiffness. The remainder of his exam is without focal neurological signs. CSF shows 100 WBC, all neutrophils (normal is less than 6 lymphocytes: there should be no neutrophils), a protein of 80 (normal is 20-45 mg/dL), and a glucose of 40 (normal is 50-80 mg/dL). What is the most likely diagnosis? A) Bacterial meningitis B) Toxoplasmosis C) Viral meningitis D) Bacterial abscess E) Herpes encephalitis

Answer A. Bacterial meningitis Explanation The results of the CSF examination indicate the likely bacterial origin. In adults, the most common cause of bacterial meningitis is Streptococcus pneumoniae. The symptoms include fever, neck stiffness, and headache. Even if treated, a bacterial meningitis can still result in neurological sequelae or death in up to 20% of cases. The diagnosis rests on the clinical symptoms in combination with a pleocytosis (consisting of mostly neutrophils), an elevated CSF protein, and decreased CSF glucose (remember to compare the CSF glucose to that of the serum). Those at most risk for developing meningitis include older adults (over 65), children (under 5), alcoholics, people with diabetes, and those who are immunocompromised. The treatment is directed to the causative organism. A CSF glucose < 34 mg/dl, a CSF to serum glucose ratio of < 0.23, a CSF protien of > 220 mg/dl, a CSF WBC of > 2000 leucocytes/mm3 or neutrophil count of > 1180 neutrophils/mm3 are all individual predictors of bacterial meningitis with > 99% certainty. CSF Gram Stain is positive in 60 - 90 % of cases of bacterial meningitis depending on the causative organism. CSF cultures is positive in 70 - 85% of bacterial meningitis. Viral meningitis presents in the same way but causes a CSF pleocytosis that is mainly lymphocytes. In viral meningitis, the CSF glucose may be low, and the CSF protein may be normal. Bacterial abscesses present with headache, fever, and focal neurological signs. Toxoplasmosis is most common in immunocompromised adults but can occur in older individuals. As with abscess, it causes focal (or multifocal) neurological deficits as well.

An 80-year-old man with HTN, BPH, and CAD develops herpes zoster involving the trigeminal nerve. After the lesions heal, he still has severe pain involving his right eye and right side of his forehead. Which of the following treatments would you recommend next? A) Gabapentin B) Phenytoin C) Amitriptyline D) Nortriptyline E) Hydrocodone/oxycodone

Answer A. Gabapentin Explanation This elderly patient is suffering from postherpetic neuralgia. Because of his history of coronary artery disease and BPH, tricyclic antidepressants (amitriptyline and nortriptyline) would be contraindicated. In patients with BPH, tricyclics can cause acute urinary retention. There is a higher risk for arrhythmia in patients with CAD treated with tricyclics. Gabapentin is effective and well tolerated for the treatment of post-herpetic neuralgia. Narcotics can be used for treatment of post-herpetic neuralgia, but a long-acting narcotic like hydrocodone/oxycodone would not be the first option. Narcotics are more toxic in the elderly, and in this case could lead to urinary retention. The anti-seizure medications gabapentin and carbamazepine are effective for post-herpetic neuralgia. Phenytoin is not usually used because of toxicity and unproven efficacy.

A 40-year-old woman comes to your office with complaints of gradually worsening memory. Her husband states that she has become irritable and "difficult to live with" over the past 9-12 months. There has been no fever, malaise, or neck stiffness. Her mother died at a young age of "something neurological." On examination, she has quick jerks in both arms and face. Which of the following is the most likely diagnosis? A) Huntington disease B) Creutzfeldt-Jacob disease C) Sydenham chorea D) Infarct of the subthalamic nucleus E) Herpes encephalitis

Answer A. Huntington disease Explanation This is an autosomal dominant illness caused by a trinucleotide repeat. Men and women are affected equally. It occurs in 5:100,000 people. It is a progressively worsening neurological illness that usually progresses to death in 15-20 years. It manifests as worsening mental status and involuntary movements (chorea = quick, jerk-like movements; and athetosis = slow, writhing, snake-like movements). Often, there are associated personality changes (irritable, argumentative, erratic, impulsive) and depression (as the person is aware of their deficits). Rigidity and dystonia usually occur late in the illness. Diagnosis is made in the setting of positive family history, clinical features, and genetic testing demonstrating the HTT gene. Atrophy of the caudate nuclei on CT or MRI ("boxcar" ventricles) is characteristic and correlates with deterioration in cognition. There is no known treatment; however, dopamine-blocking agents such as haloperidol can help to manage the abnormal movements. Sydenham's usually affects children and adolescents and is post-infectious, due to a group A hemolytic streptococcal infection or acute rheumatic fever. Herpes can cause rapid onset of altered mentation and personality changes and is associated with fever and neck stiffness. CJD is described in a separate question (Oops! I gave you a hint, if you haven't done this question yet). An infarct of the subthalamic nucleus causes hemiballismus—quick flinging movements of the limbs contralateral to the side of the infarct. It occurs in older adults and is due to the same causes of ischemic or hemorrhagic stroke.

A 78-year-old with hypertension and diabetes is admitted to the emergency department due to sudden onset of double vision earlier the same day. He denies troubles with weakness, sensory loss, or troubles with attention, concentration, thinking, or memory. On exam, he is unable to adduct the right eye. What is the correct diagnosis? A) Internuclear ophthalmoplegia B) Left 6 th nerve C) Right 6 th nerve palsy D) Cerebral aneurysm

Answer A. Internuclear ophthalmoplegia Explanation Internuclear ophthalmoplegia (INO) is often due to a small stroke involving the ipsilateral medial longitudinal fasciculus. The most common causes of stroke in this case are poorly controlled high blood pressure or poorly controlled diabetes. In INO, the stroke causes extraocular muscle weakness (ophthalmoparesis), which is the reason that one eye cannot fully adduct. The partner eye diverges from the affected eye during abduction, producing horizontal diplopia. In other words, if the right eye is affected, the patient will "see double" when looking to the left. The diplopia produces images that are side-by-side. This could not be a 6 th nerve problem: The 6 th nerve causes the eye to abduct (thus the name of the nerve, abducens). An aneurysm may cause diplopia but often causes other neurological symptoms, which he does not describe.

A 78-year-old man is brought in to your office by his daughter for evaluation. She was recently told by the patient that he converses with his wife for the last year, despite the fact that his wife died 5 years ago. She notes that he seems to have difficulty accomplishing tasks he could easily accomplish before. For example, he could not remember how to shave once last week and called his daughter for help. She notes that on occasion he seems unable to care for himself and requires assistance from his family to perform some activities of daily living, but seems to function well the following day. The patient remarks that he enjoys talking with his wife and children as often as he can. Physical exam reveals a thin older man in no distress. He is alert and oriented to person and place but cannot state the correct date. Neurologic exam reveals normal strength and reflexes. Gait is somewhat slowed. Mental status examination reveals impairment in drawing a clock and overlapping pentagons. Which of the following is the most likely diagnosis? A) Lewy body dementia B) Alzheimer dementia C) Major depressive disorder D) Normal pressure hydrocephalus E) Delirium

Answer A. Lewy body dementia Explanation This patient demonstrates a history of progressive cognitive decline consistent with dementia. The core clinical features of Lewy body dementia are fluctuations in cognition, visual hallucinations, and parkinsonian features. This patient has 2 of the 3 features. Parkinsonian symptoms typically present after onset of cognitive impairment. Alzheimer dementia typically involves more significant impairments in short-term memory as the primary cognitive deficit. Delirium is characterized by fluctuating levels of consciousness that fluctuate over hours to days and would not fit the time course described in this patient. Normal pressure hydrocephalus is characterized by dementia, urinary incontinence, and ataxic gait. Major depressive disorder would be on the differential of this patient as it can cause psychotic features when severe and can cause cognitive impairment in the elderly. However, the diagnosis requires the typical features of depression—depressed mood and/or anhedonia. The patient in this vignette is not described with either.

A 70-year-old female complains of pain in neck, shoulders, hips, and thighs x 2 months. She has not been sleeping well and has noticed a 10-lb weight loss in 3 months. Exam: normal strength. Labs: WBC 9,500, normal differential Hgb 10.5 Platelets 450,000 Na 145 K 4.2 HCO 3 24 BUN 12 Creat 0.6 Glu 92 CPK 70 (50-170) Which of the following is the most appropriate next step in management? A) Low-dose prednisone daily. B) Peroneal nerve biopsy. C) Start pregabalin. D) Acetylcholine receptor antibodies. E) Anti-Jo antibodies.

Answer A. Low-dose prednisone daily. Explanation We snuck in a rheumatologic disease here to illustrate that overconfidence in an area is a common problem with Board preparation. Overconfidence about your knowledge can arise when you see questions in a particular context and subsequently answer them correctly as a result (e.g., answering neuro questions correctly because you know you're studying neuro). On the Board exam, the questions are not identified by section. Differentiating rheum conditions from neurologic conditions can be a real challenge. Regardless of what section you're studying, get in the habit of constructing a problem list and looking for patterns/illness scripts. The problem list for this patient: Pain (not weakness!) in the shoulders, hips, and thighs in an elderly woman causing sleep disturbance 10-pound weight loss Anemia and thrombocytosis Normal examination Normal CPK For what illness does this script fit? Polymyalgia rheumatica. The options ask you to choose from myasthenia gravis (acetylcholine receptor antibodies), a myositis (anti-Jo antibodies associated with polymyositis; peroneal nerve biopsies help diagnose confusing cases of myositis), and empiric treatment for fibromyalgia or polymyalgia rheumatica (PMR). Be prepared to see PMR presented on the exam and juxtaposed among these diseases in the differential diagnosis. This case is not myasthenia because the presentation is pain, not weakness, and the patient profile is an elderly female. Any shoulder pain in an elderly female should immediately suggest PMR. Myasthenia classically worsens with use and improves with rest. Anti-acetylcholine receptor antibodies are present in most cases of myasthenia and can be used to diagnose the condition (replacing the Tensilon® test). Myositis should be considered, but the normal CPK excludes it on the Boards. The only disease involving muscle degeneration that has a normal CPK, on the Boards, is a steroid myopathy. Fibromyalgia is a disease associated with abnormal sleep patterns and diffuse body aches; but on the Board exam, fibromyalgia will probably never be the right answer for any complaints in an elderly woman. This woman also has danger signs: anemia, thrombocytosis, and weight loss. These signs and symptoms suggest a true underlying serious disease process, such as a vasculitis, and are not associated with fibromyalgia. In practice, you might perform other tests (such as a TB skin test) before instituting systemic corticosteroids. You might even refer this patient to a rheumatologist because long-term use of systemic steroids in an elderly woman is a rather serious treatment if you aren't completely comfortable with the diagnosis. The Board exam, however, may ask you to commit to empiric treatment before you are necessarily comfortable with a diagnosis. If empiric treatment is the best answer out of the choices listed, then don't fret about it. It's not a real patient; it's just a Board question. Choose the best possible answer and move on.

A 34-year-old woman is referred to your office because of weakness. The problem has been gradually worsening for 10 days. She has no pain, denies trauma, and has had no recent illnesses or fever. She has no other medical illnesses. In college, she said that she had transient blurriness in one eye. This lasted "a few days," and she cannot recall which side was involved. She did not see a physician for the visual symptoms because they resolved spontaneously. Physical exam: She has a pale, mildly atrophied optic disc on the right. She has 4/5 strength in the left upper and lower extremity, brisk reflexes in the left arm and leg, and an upgoing toe on the left. Tone is normal. Strength and reflexes on the right are normal. Testing of sensation is normal. Coordination is remarkable for slowed rapid alternating movements in the left upper extremity. Her gait is slow. Which of the following is the most likely diagnosis? A) Multiple sclerosis B) Transverse myelitis C) Stroke D) Cauda Equina syndrome E) Guillain-Barré syndrome

Answer A. Multiple sclerosis Explanation The most likely diagnosis is multiple sclerosis (MS). The history of transient neurologic dysfunction (the blurred vision in college) strongly supports the diagnosis. The patient has an upper motor neuron disease (the increased reflexes and positive Babinski), so Cauda Equina syndrome and Guillain-Barré are not possible. The symptoms involve only the left side, so transverse myelitis, which would cause a paraparesis, is not possible. Stroke is unlikely because the patient is young, without risk factors for stroke, and the symptoms have gradually worsened for 10 days. She denies orthostatic hypotension.

A 33-year-old woman is referred for episodic dizziness. When you see her, she reports a spinning sensation that is accompanied by nausea, and less often, vomiting. The dizziness is episodic, usually lasting 45 minutes. It first started 4 years ago. She reports ringing in the left ear and has more recently noticed hearing loss in the same ear (left) that is unassociated with the attacks of dizziness. There have been no recent illnesses, fever, chills, or focal neurologic deficits. Exam: The patient's general exam is negative. Neurological exam reveals mild hearing loss to the tuning fork, but is otherwise unremarkable. Which of the following is the most likely diagnosis? A) Ménière disease B) Stroke C) Labyrinthitis D) Benign positional vertigo E) Vestibular neuronitis

Answer A. Ménière disease Explanation The triad of episodic vertigo, hearing loss, and tinnitus makes this the most likely diagnosis. In benign positional vertigo, the person would not be expected to have hearing loss. Labyrinthitis and neuronitis are usually monophasic illnesses, likely due to a virus. Although stroke can cause vertigo, it is unlikely to be recurrent. In addition, stroke rarely causes vertigo by itself. The hearing loss in Ménière disease is usually progressive.

A 25-year-old man comes to your office complaining of an excruciating unilateral headache for the past week. He says, "It's like an ice-pick behind my eye." You discover that it occurs 8 times per day, and that each headache lasts 30 minutes. The headache is associated with eye watering on the same side. He had headaches like this 1 year ago. They lasted 6 weeks before resolving spontaneously. His dad gets the same kind of headache. PHYSICAL EXAMAMINATION: His general and neurologic exams are normal. Which of the following treatments is the most effective at aborting his symptoms? A) Oxygen B) Prednisone C) Verapamil D) Valproic acid E) NSAIDs

Answer A. Oxygen Explanation Intranasal oxygen therapy is the most effective abortive remedy. Delivered for about 15-20 mins. If oxygen is not available, triptans can be used to abort the headache. Verapramil, prednisone, topiramate, and lithium can be used for prophylactic treatment.

A 23-year-old woman comes to your office complaining of a unilateral throbbing headache that was preceded by "seeing flashing lights." The headache worsened over a 60-minute period, has been present for three hours, and seems to be getting better. The headache is severe, a "9/10." She is nauseous and had vomited. She says that the headache is worse if she sees bright lights or hears loud noises. She has had this type of headache since her early teens and has them two to four times per year. On examination, she has no weakness or numbness. There is no papilledema on funduscopic examination. Serum tests are normal. Which of the following would you recommend for treatment? A) Sumatriptan B) Propranolol C) Amitriptyline D) Divalproex sodium/valproate E)Verapamil

Answer A. Sumatriptan Explanation She has the classic signs and symptoms of migraines. Since she has headaches infrequently, she would benefit more from an abortive agent as opposed to prophylaxis (valproate, propranolol, verapamil, and amitriptyline all fall in this category of treatment options). Verapamil is perhaps more effective in cluster headache, a type of headache that affects men more than women.

A 27-year-old woman is brought to the emergency department by EMS after being found down by a neighbor. The neighbor states the woman did complain of a headache with some nausea but otherwise seemed fine. The patient does not use drugs, but she does drink wine daily and smokes cigarettes. She has no past medical history and takes no medications. Exam: T 98° F, RR 8, BP 100/65, HR 100 BMI 19, oxygen saturation 99% No response to sternal rub Pupils equal and reactive "Doll's eyes" present Normal tone; no spontaneous movements Toes down-going ABG: pH 7.30, pO2 80, pCO2 60 Na 137, K 3.5, Cl 104, HCO3 27, BUN 25, Creat 0.7, Glu 86 Which of the following is the most appropriate next step in patient care? A) Parenteral hydrocortisone tid B) Carboxyhemoglobin level C) Ceftriaxone 2 grams intravenously D) Nasogastric tube and n-acetylcysteine E) Naloxone

Answer B. Carboxyhemoglobin level Explanation These questions that require interpretation of history, exam, and electrolytes. In this question, an otherwise healthy woman is found down. She had a little headache and some nausea; then, all of a sudden, she's comatose. This is the sort of presentation you should associate with carbon monoxide poisoning. Notice the question gives you no clue as to how the patient is exposed to the CO. There's no discussion of generators or exhaust fumes or kerosene or wintertime heat. Her exam shows you a comatose patient with an intact brain stem. Her blood gas shows a respiratory acidosis with a normal pO2. And her oxygen saturation is low-normal. Remember that CO poisoning with development of carboxyhemoglobin is often associated with normal pO2 and very high pulse oximetry values. The pO2 is normal because the test measures dissolved O2 in the blood which is not affected much by CO. The O2 saturation is very high because the pulse oximeter does not distinguish between oxyHgb and carboxyHgb. PCO2 = 60; the acidosis is due to hypoventilation. The anion gap is normal (137 - 104 - 27 = 6). The BUN:Cr is increased (35), reflecting a prerenal state. Usually something in the case points you toward CO poisoning and, in this instance, it is the preceding headache and the nausea. But remember that any presentation of a patient who developed acute coma should trigger the need to rule out CO poisoning. And hypoventilation with normal pO2 and very high O2 sat makes the diagnosis likely. Look out for questions that also include hypovolemic hyponatremia as a feature. If the sodium is low, you have to also consider adrenal insufficiency and myxedema coma. Naloxone is a common distractor in exam questions about coma. This is generally not the correct answer unless you know for certain the person overdosed on a narcotic. The option for n-acetylcysteine is realistic if the question gave you some indication of acetaminophen overdose several days ago. If there is an indication of an infection such as bacterial meningitis, antibiotics may be indicated—but there is none

A 25-year-old man comes to your office complaining of an excruciating unilateral headache for the past week. He says, "It's like an ice-pick behind my eye." You discover that it occurs 8 times per day, and that each headache lasts 30 minutes. The headache is associated with eye watering on the same side. He had headaches like this 1 year ago. They lasted 6 weeks before resolving spontaneously. His dad gets the same kind of headache. PHYSICAL EXAMINATION: His general and neurologic exams are normal. Which of the following is the most likely diagnosis? A) Migraine headache B) Cluster headache C) Subarachnoid hemorrhage D) Tension headache E) Pseudotumor cerebri

Answer B. Cluster headache Explanation The combination of a normal exam with these headache symptoms describes a cluster headache. For the exam, you must know the differences among each of the listed headaches. Cluster headaches are more common in males and respond to 100% oxygen.

A 49-year-old publishing magnate presents with complaint of severe stabbing headaches. "It feels like an ice-pick in my skull." When he gets the headaches, he also notes occasional lacrimation and nasal congestion. His headaches develop over 5-10 minutes and then last from 20 minutes to 2 hours. He reports lately he has had them 2-3 times daily and had similar episodes last year that lasted 8 weeks. These episodes seem to occur around a Board review conference. What type of headaches does he seem to have? A) Migraine headache B) Cluster headache C) Headache due to pseudotumor cerebri D) Tension headache E) Simple headache

Answer B. Cluster headache Explanation He has classic cluster headaches. The stabbing retroorbital pain described as an ice pick is classic. Cluster headaches are associated with rhinorrhea, nasal congestion, lacrimation, or conjunctiva changes. Cluster headaches are more common in males. They can be quick or gradual in onset and may last 20 minutes to 2 hours in duration. The clusters usually last 6-12 weeks. Verapamil, prednisone, and valproate have been used as preventative measures. 5-HT agonists and ergotamines have been used to abort the headaches.

A 45-year-old woman comes to her clinic appointment with increasing symptoms of confusion over the past day, sweating, and increasing anxiety. She fell roller-skating yesterday and severely injured her shoulder (her R arm is in a sling). She has a history of depression, GERD, hypertension, and headaches. Meds: omeprazole, lisinopril, metoprolol, citalopram, tramadol, and sumatriptan prn. PE: VS: BP 160/100, P 100 Tremor present, muscle rigidity What is the most appropriate treatment? A) Discontinue metoprolol. B) Discontinue citalopram and tramadol. C) Mannitol. D) Dantrolene. E) IVF.

Answer B. Discontinue citalopram and tramadol. Explanation This patient has serotonin syndrome. It was provoked by the addition of tramadol to her baseline SSRI. She may also have used sumatriptan, a medication on her med list. The best treatment would be to stop the drugs causing the syndrome. A benzodiazepine might help the hypertension and tachycardia.

A 56-year-old man presents for routine follow-up and complains of one month of increasing stiffness and trouble getting started walking. ROS is otherwise negative. Medical history includes DM, HTN, and newly diagnosed AIDS, which was diagnosed after an episode of psychosis 3 months ago. Meds: AZT/3TC, efavirenz, reserpine, glipizide, HCTZ Exam: Normal except for bilateral axillary lymphadenopathy, which is improving. No increased motor tone or rigidity. No tremor. Gait is slightly wide-based, and he has trouble starting from a stopped position. Cerebellar exam is normal. Which of the following is the most appropriate next step in management? A) Referral to a neurologist. B) Discontinue reserpine. C) Lumbar puncture and send CSF for JC virus PCR. D) Change efavirenz to nevirapine. E) Start carbidopa/levodopa.

Answer B. Discontinue reserpine. Explanation This case illustrates something called the "Oh, no, the patient's on AIDS drugs!" phenomenon. If you're the typical non-ID physician, you probably started to hyperventilate as soon as you saw the list of drugs, beginning with AZT. Then you probably focused on the AIDS drugs and assumed that the question somehow related to them. This happens because the AIDS drugs are so complicated and diverse. Number one priority in dealing with a question about an AIDS patient: Don't panic. Number two: Look for what you know and realize you are sitting for General Medicine Boards, not ID Boards. What are they asking you about this patient that relates to General IM? This patient is presenting with parkinsonian symptoms: Wide-based gait Subjective stiffness Slow motion Yet, he doesn't have hypertonicity or cog-wheeling. So is this Parkinson disease? Remember: You can't diagnose Parkinson's if the patient is on a drug that commonly causes a movement disorder. You have to stop the drug first. This patient is taking reserpine, a drug commonly associated with bradykinesia. Others include: neuroleptics (like haloperidol), prochlorperazine, and metoclopramide. Carbidopa/levodopa is for true Parkinson disease. HAART drugs are not associated with movement disorders. JC virus causes PML in AIDS patients, but PML does not present as a movement disorder. You do not need to refer a patient to a neurologist if you can simply stop a drug and make him better. And the Boards frequently test whether you recognize drug side effects.

A 27-year-old female complains of progressive weakness in the lower extremities x 2 weeks. History of self-limited gastroenteritis 1 month ago Afebrile Normal cranial nerves Motor weakness in hip and knee flexors Decreased deep tendon reflexes Normal sensation Which of the following laboratory results is the most consistent with the correct diagnosis? A) Positive Western blot for Borrelia burgdorferi B) Elevation of protein in the CSF C) Finding acetylcholine receptor antibodies in the serum D) Positive serum RPR E) Positive CSF PCR for Borrelia burgdorferi

Answer B. Elevation of protein in the CSF Explanation The illness script presented in this case is progressive weakness following diarrhea. That is the script for Guillain-Barré (GB). Specifically, Campylobacter is often the inciting etiology. Patients with GB are typically afebrile and have only motor weakness and loss of reflexes. Sensory is typically intact. (We know that in real life, patients can have strange sensory presentations. But this isn't real life! It's the Boards!) The Boards frequently test your knowledge of how a lumbar puncture contributes to the diagnosis of GB. You should know that the CSF in GB has a normal cell count but is associated with a high protein. Acetylcholine receptor antibodies are seen in the serum of patients with myasthenia gravis, which has a completely different presentation (exertional weakness [not paralysis] that improves with rest). CNS Lyme disease can rarely present similarly, but PCR of the CSF is not useful in the diagnosis of neuroborreliosis. Lyme diagnosis requires more than a positive Western blot; and of the two (GB or a rare presentation of Lyme), GB is the most likely diagnosis. Tertiary syphilis (tabes dorsalis) is a potential cause, but this 27-year-old woman is very unlikely to have tertiary syphilis, given her young age. GB is commonly associated with antecedent Campylobacter infections.

A 45-year-old male patient of yours calls you on a Sunday afternoon. He has been having nausea, vomiting, and diarrhea today. He also has had a headache and mild dyspnea. These symptoms actually improved when he was shoveling snow earlier in the day but have now returned. He reports his wife is sick in bed with the same "flu-like" symptoms. Which of the following do you recommend next? A) Have the patient see you in the clinic tomorrow if the symptoms persist. B) Have the patient call 911 for emergency evaluation/transport to emergency department. C) Prescribe amantadine 100 mg bid x 7d for both the patient and his wife. D) You will make a house call today.

Answer B. Have the patient call 911 for emergency evaluation/transport to emergency department. Explanation This patient has symptoms consistent with CO poisoning: headache, nausea, vomiting, diarrhea, and dyspnea. The key piece of clinical information is the improvement when he went outside to shovel snow. The appropriate management would be immediate removal from his home and immediate supportive care in an emergency department. Remember that the SpO2 and PO2 will be normal. Indications for hyperbaric oxygenation include coma, pregnancy, acidosis and a CO level above 30%. Hyperbaric oxygenation reduces the half life of CO from 300 mins (on room air) to 25-30 minutes on HBOT.

A 34-year-old woman is being treated with carbamazepine for her partial seizure disorder. (She has simple partial and complex partial seizures due to a past head injury.) She is now considering becoming pregnant. Which antiseizure medication would be an appropriate choice in order to maintain seizure control through her pregnancy? A) Valproate. B) Lamotrigine. C) Phenytoin. D) None of these—stop her medications. E) Phenobarbital.

Answer B. Lamotrigine. Explanation This is a difficult clinical situation. There is no antiseizure medication which is known to be completely safe during pregnancy. There is much controversy as to what is the "best" answer. However, phenobarbital, valproate, and phenytoin are all pregnancy class "D," which means that there is a known risk of birth defects when used during pregnancy. Further, recent studies have shown that phenobarbital administration during pregnancy reduces the child's IQ. Valproate stands out as causing the highest percentage of birth defects (and, therefore, should be avoided, if possible). Lamotrigine has been reported to cause a low incidence of cleft lip and palate; however, it otherwise seems to be relatively safe during pregnancy. However, the serum levels often decrease, and the dose must be adjusted upward during the pregnancy to maintain consistent serum levels (levels are often checked monthly, with subsequent dose adjustments). Stopping medications presents a problem as well—seizures that occur during pregnancy are injurious to the developing baby and need to be controlled. In short: Use the lowest effective dose of medication (ideally monotherapy) before pregnancy begins and maintain this through the pregnancy. Also, be sure to give folate!!!

A 48-year-old female presents to your office complaining of pain in her back and buttocks x 3 months. The pain is a severe ache, noticeable most often when she stands for prolonged periods. Pain is relieved when she sits or performs her daily exercise on an inclined treadmill. Exam is normal. Which of the following is the most appropriate next step in management? A) Epidural steroid injection and bed rest B) MRI of the spine C) Lower-extremity vascular evaluation D) Analgesics prn E) Referral to a neurosurgeon for laminectomy

Answer B. MRI of the spine Explanation When approaching back pain questions, consider how the various forms of back pain present. The important features in this case: Radiation to buttocks Worse with standing Relieved with sitting and bending (walking on an incline) Disk herniation is worse with sitting. Back strains and ischemic claudication features do not change with posture. This presentation is most consistent with spinal stenosis (neurogenic claudication). Spinal stenosis is diagnosed by MRI (if the patient is a candidate for surgery or epidural steroid injections, per ACP recommendations from October 2007) and treated conservatively. Diagnosis should be pursued first with treatment options employed after you know what is causing the pain (injections, analgesics, laminectomy). Lower-extremity vascular evaluation is not necessary because she has no symptoms of insufficiency.

A 19-year-old woman comes to your office complaining of frequent severe headaches, usually on the right. They are usually unilateral, throbbing, and associated with photophobia and phonophobia. She denies associated rhinorrhea, watering eyes, or conjunctival erythema. Often, these will worsen at the time of her menses. She has had them for about 1 year. They began with a frequency of one per month, but now occur about two times a month. The headaches last 3-4 hours. She denies associated neurologic problems such as weakness or numbness. So far, she has taken only over-the-counter medications like acetaminophen and ibuprofen. Both medicines will partially relieve her headache. There is a positive family history (in her mother and a maternal aunt) of similar headaches. Physical Exam: She is average weight, and her general physical exam, neurological exam, and routine laboratories are normal. Which of the following is the most likely diagnosis? A) Cluster headache B) Migraine headache C) Intracranial tumor D) Pseudotumor cerebri E) Tension headache

Answer B. Migraine headache Explanation Several historical factors support this: a positive family history, unilateral throbbing, provocation by menses, and the lack of other neurologic complaints. Other features that sometimes support the diagnosis of migraine are: age of onset (usually in the second or third decade of life), sex (more common in women), and the association with both phonophobia and photophobia. Although these complaints are not unique to migraine, they occur commonly. Some patients with migraine (~25%) will experience an "aura" that consists of flashing lights that precedes the onset of pain. This type of aura is also called "scintillating scotomata." Auras last a minimum of 5 minutes and a maximum of 60 minutes. Of all patients that present to primary care providers or EDs with recurrent headaches, > 90% will have migraines. The prevalence of migraines is about 12% in the general population. Cluster headache is similar to migraine in that it is also usually unilateral. However, cluster headache often occurs in young men. A cluster headache is usually described as a "hot poker behind the eye," has a duration that is measured in minutes rather than hours (typically 20-30 minutes), and, of course, clusters. Typically, the headaches will occur for a period of several weeks. The cluster headaches may occur around the same time of day and may occur at the same time of year in some patients. A positive family history of similar headaches would support a diagnosis of cluster headache. Tension headaches are "bandlike," "squeezing," "dull," and "constant" in character. They have variable frequency and can last many hours to several days, weeks, months, or even years. Pseudotumor typically affects young, obese women. The headache is dull and constant, and papilledema (usually bilateral) would be present on examination. Intracranial tumors typically cause focal neurologic deficits. Whenever you hear about headache and focal neurologic deficits, think about the possibility of an intracranial mass lesion. If the patient also has a fever, think of abscess.

A 68-year-old man is brought to your office by a family member (his daughter) due to a gradually worsening problem with short-term memory over the past 2-3 years. Further questioning identifies worsening urinary incontinence as well. His exam shows a mildly disheveled man in no distress. He has no focal neurological deficits. The funduscopic exam is negative for papilledema. His gait is magnetic. Which of the following is the most likely diagnosis? A) Alzheimer's dementia B) Normal pressure hydrocephalus (NPH) C) Parkinson's disease D) Creutzfeldt-Jacob disease E) Multi-infarct dementia

Answer B. Normal pressure hydrocephalus (NPH) Explanation The question highlights a person with a gradually worsening dementia, which eliminates CJD from the differential. Although Alzheimer's and multi-infarct dementia can cause memory problems, the triad of gait abnormalities (magnetic gait), incontinence, and dementia is "classic" for NPH. NPH can be idiopathic or acquired following intraventricular or sub arachnoid hemorrhage, acute or chronic meningitis. In this illness, LP is diagnostic, and can also be therapeutic. Some people will experience an improvement in their mental status temporarily after removal of a large volume of CSF. Usually, it is this test in combination with an MRI that strongly supports the clinical history, thereby making the diagnosis of NPH. Parkinson's does not have a magnetic gait.

A 27-year-old male with history of IDDM and seizure disorder presents with a 3-week history of fatigue, low-grade fevers, and diffuse lymphadenopathy. Exam is remarkable for diffuse lymphadenopathy (cervical, axilla, inguinal; nodes 2-3 cm in each area), tender liver, and fever of 101° F. Meds: Phenobarbital, phenytoin, cisapride, and nortriptyline. Labs: WBC 9.9, Hct 38, SGOT 90, SGPT 78, BUN 20, Cr 1.1, Glu 236, RPR negative. Which of the following diagnoses is most likely? A) Hodgkin disease B) Reaction to phenytoin C) Syphilis D) Hepatitis B E) Tularemia

Answer B. Reaction to phenytoin Explanation This patient is having a reaction to phenytoin, creating pseudolymphoma syndrome. He has fever, elevated transaminases, and generalized lymphadenopathy. A negative RPR effectively rules out secondary syphilis. He has no exposure history to suggest hepatitis B or tularemia. Hodgkin disease is a possibility, but the involvement of the liver with diffuse lymphadenopathy would indicate advanced disease, which would be unlikely to occur so rapidly.

A 64-year-old female with chronic Parkinson disease is brought to the emergency department by her family because of an acute deterioration in her condition. She is confused, agitated, and diaphoretic. Her vitals include a temperature of 101.8° F (38.8° C) and a blood pressure of 192/108 mmHg. Remarkable physical findings include dilated pupils, hyperactive bowel sounds, and myoclonus with exaggerated reflexes. Review of her medications includes a combination carbidopa/levodopa preparation, selegiline, amantadine, and fluoxetine, which were recently prescribed by her primary care provider. She has had no recent surgeries and has been fundamentally homebound. Which of the following is the most likely cause of this presentation? A) Anti-cholinergic toxicity B) Serotonin syndrome C) Malignant hyperthermia D) Neuroleptic malignant syndrome

Answer B. Serotonin syndrome Explanation The combination of a serotonin reuptake inhibitor (fluoxetine) with a monoamine oxidase inhibitor (selegiline) leads to increased levels of serotonin, and, with high enough levels, this can manifest as serotonin syndrome. The syndrome is comprised of mental status compromise (confusion, agitation, hallucinations), autonomic dysfunction (diaphoresis, dilated pupils, hypertension, hyperthermia, increased bowel activity), and neuromuscular abnormalities (hyperreflexia, myoclonus, muscle rigidity). Malignant hyperthermia occurs as a result of inhaled agents used to induce surgical anesthesia. Anticholinergic toxicity varies from this presentation as a patient with hot, dry skin and absent bowel activity with normal reflexes. Neuroleptic malignant syndrome can closely resemble hyperserotonergic syndrome but manifests with "lead pipe" rigidity and bradykinesia. The review of the medication list can help distinguish the relative risk of the two disorders, but it should be noted that an abrupt decrease in L-dopa dosing can precipitate the symptoms of neuroleptic malignant syndrome.

A 43-year-old woman is brought to the emergency department by her husband. She had a right middle cranial fossa meningioma removed less than 2 weeks ago. She went home on postoperative day number 5. Four days ago, the staples were removed in the neurosurgeon's office. Ever since the surgery, she has had headaches. She was given acetaminophen with codeine, which initially helped. However, the headaches have worsened over the past 3 days. In addition, she has had a low-grade fever to 100.5° F. Exam: The surgical wound looks clean (no drainage, fluctuance, or excessive redness). Chest is clear to auscultation. The patient appears slightly sleepy. She does not have signs of meningeal irritation. Cranial nerves are normal. Speech and language are normal. There is mild weakness (4/5) in the left arm. There is a left pronator drift. Reflexes are mildly increased in the left arm. Sensation and coordination appear intact. Which of the following is the most likely diagnosis? A) Opiate intoxication B) Ischemic infarct C) Abscess D) Viral encephalitis E) Hemorrhagic infarct

Answer C. Abscess Explanation In this case, the gradually worsening headaches after an invasive procedure strongly suggest a post-operative abscess. The fever and focal neurological deficits confirm this. Remember, the triad of fever, focal deficits, and headache means abscess. Her pupils are normal and, with the focal findings, this excludes opiate intoxication.

50-year-old man comes to the office with "clumsiness." This has been getting worse gradually over the past few weeks. At first, it involved the right foot. Later, he noticed problems in both feet and was "tripping over" his own feet. In the past week, he has noticed a change in his voice: It now sounds more "nasal." He denies recent infections. He has had no fever, chills, nausea, or vomiting. On examination, he has a "nasal" sounding voice, but his speech is fluent. Naming and repetition are excellent. He has weakness of the legs more so than the arms. He has hyperreflexia. Which of the following is the most likely diagnosis? A) Herpes encephalitis B) Brain tumor in the dominant hemisphere C) Amyotrophic lateral sclerosis (ALS or Lou Gehrig disease) D) Huntington disease

Answer C. Amyotrophic lateral sclerosis (ALS or Lou Gehrig disease) Explanation Amyotrophic lateral sclerosis affects people in their 40s to 60s. It usually causes weakness or clumsiness of the limbs first (in 75% of cases, the limbs are affected first) and then affects the bulbar muscles (20%) which are the muscles that help to coordinate speech and swallowing. When it begins, it is usually asymmetric, later affecting both sides. The hallmark of ALS is marked, simultaneous upper and lower motor neuron signs. Men are affected more often than women. Other causes of simultaneous LMN and UMN findings include: •ALS (motor only, ie no sensory loss)) •B12 def (motor + sensory) •Cervical myelopathy (motor + sensory) •Syringomyelia (LMN = arms UMN = legs, motor + sensory) •Friedreich ataxia (ataxia + motor + sensory) •Syphilis (motor + sensory) •Hyperthyroidism (motor + sensory) Huntington's does affect people in their 40s but usually causes dystonia or abnormal movements of the limbs (like choreiform, which are jerk-like, or athetoid, which are writhing movements). Huntington's also often causes anxiety, depression, or mood changes. One similarity between the two is that both can progress quickly. However, Huntington's would not cause hyperreflexia. A brain tumor would most likely cause gradually worsening focal neurological deficits like weakness, numbness, and hyperreflexia. However, the signs would be contralateral to the hemisphere in which the tumor was enlarging and would not be bilateral. A tumor in the dominant hemisphere could affect language but would likely produce an aphasia, which would manifest as troubles with naming, repetition, or production of speech (he has none of these findings). The one instance where a tumor could affect both sides of the body, as well as speech, would be in the brainstem. In this instance, he would likely also have one or more cranial nerve deficits. Herpes encephalitis most often presents with fever, personality changes, etc.

A 56-year-old comatose obese woman is brought to the ED by her husband. She has had several days of headache and has been "sleepy." On the day of admission, she became unresponsive. On exam, she has hyperreflexia on the left (arm and leg), spasticity in the left upper and lower extremity, and seems to withdraw from painful stimuli, less well on the left. There is no atrophy or fasciculations of the muscles in the left arm or leg. Funduscopic exam shows bilateral papilledema; the right pupil is large and unreactive, while the left is midpoint and reacts to light. What is the most likely cause of her coma? A) Pseudotumor cerebri B) Ketoacidosis C) CNS tumor D) Hypercalcemia E) Normal pressure hydrocephalus

Answer C. CNS tumor Explanation Specifically, a right hemisphere CNS tumor. All of the choices, except for normal pressure hydrocephalus, can cause coma. The patient has upper motor neuron signs (hyperreflexia and spasticity), which suggest pyramidal (long tract) involvement in the right hemisphere. The enlarged right pupil is often referred to as a "blown pupil." It is caused by pressure that is directly placed on the third cranial nerve due to uncal herniation. Weber syndrome refers to ipsilateral third nerve palsy and contralateral hemiparesis: it can be caused by tumors, infarction or bleeds, as well as demyelination. Pseudotumor cerebri occur in young, obese women who constitute 90% of all cases. It is thought to be due to increased estrogens (it has been associated with the use of oral contraceptives); however, the number of possible causes is quite long. Normal pressure hydrocephalus occurs more often in older adults. It presents with the triad of ataxia, urinary incontinence, and dementia. The cause is not known, though many have proposed that it is due to an abnormality in cerebrospinal fluid production or reabsorption. Her neurologic findings are not consistent with ketoacidosis or hypercalcemia.

A 46-year-old man develops a severe headache this morning while working at his car dealership. He also has nausea and has had 1 episode of emesis. He has shoulder and neck stiffness that developed today. He denies any history of migraine headaches or any similar headaches. Which of the following diagnostic tests should be ordered next? A) CT scan with contrast B) Lumbar puncture C) CT scan without contrast D) MRI scan

Answer C. CT scan without contrast Explanation This patient has symptoms consistent with acute subarachnoid hemorrhage. He is at the average age (45) for this to occur, has sudden onset of a severe headache, and no prior history of severe headaches. The neck stiffness is due to meningeal irritation from blood. Blood in the subarachnoid space shows up very well on CT scan, and contrast is not needed. Acute bleeds are better imaged by non-contrast CT scan than with MRIs. The CT scan is a preferable first option over a lumbar puncture, because if the CT is positive, an LP would not be necessary. The non-contrast CT has a sensitivity of about 95% in the first 24 hours of symptoms, the sensitivity drops to about 50% at day 7. If the CT is negative, then an LP should be done looking for xanthochromia, which occurs 6 hours after SAH and can last for up to 2-4 weeks.

A 76-year-old man with a history of hypertension and CABG surgery 5 years ago is brought to the ED with a sudden onset of dizziness and clumsiness. He feels mildly nauseated, but has had no vomiting. He has been in relative good health recently. He denies malaise, fever, and chills. Exam: His general exam is normal. His vital signs show an elevated BP of 170/90. Neurological exam shows direction-changing nystagmus, and ataxia of both the left arm and leg. Otherwise, his exam is normal. Which of the following is the most likely diagnosis? A) Ménière's B) Benign positional vertigo C) Cerebellar infarct D) Vestibular neuronitis E) Labyrinthitis

Answer C. Cerebellar infarct Explanation Infarct is most likely to explain the sudden onset of this constellation of signs and symptoms. Cerebellar infarct is very dangerous, and anyone with this diagnosis should be observed closely. Maximal swelling may occur up to 96 hours post infarction. As swelling occurs, there is a risk of compression of the brainstem. If the brainstem is at risk, surgery may be necessary as a life-saving procedure. Nystagmus and ataxia would not occur with BPV or Ménière disease.

A 62-year-old man is brought to the ED due to worsening mental status. When the family is questioned, the family describes a worsening of cognition over the past 4-6 weeks. As he has worsened, he has had quick jerk-like movements of the extremities. On examination, he has myoclonus in both arms (the jerks); these are precipitated by sudden noises. In addition, he has bilateral spasticity, and hyperreflexia. He is quite confused and is oriented to person and month only. What is the most likely diagnosis? A) Alzheimer dementia B) Pick disease C) Creutzfeldt-Jacob disease (CJD) D) Huntington disease E) Hepatic encephalopathy

Answer C. Creutzfeldt-Jacob disease (CJD) Explanation The sporadic form of CJD is the most common spongiform encephalopathy to occur in humans and is caused by a transmissible protein particle called a prion (there is no DNA or RNA). Its incidence is 1:1,000,000. It causes a rapidly progressive dementia that is associated with myoclonus (which is usually precipitated by sudden noises or startle). Pyramidal, extrapyramidal, and cerebellar signs are also common. The test that is most often performed to confirm the diagnosis is an EEG, which will show characteristic 1-2 Hz sharp discharges on a slowed background. Brain MRI also shows some suggestive findings. Detection of CSF 14-3-3 protein has a 92% sensitivity and a 80% specificity for CJD. False positives 14-3-3 in CSf occur in CNS metastatic disease, HSV encephalitis and hypoxic encephalopathy. In CJD death occurs in about 8 months. There is no known treatment. Alzheimer's and Pick's present in nearly identical ways and cause a dementia that is very gradual in onset (usually over years). Although hepatic encephalopathy can be rapid in onset, the physical findings usually include asterixis, not myoclonus. Huntington's also causes a dementia, usually in a younger person (35 years or older). It is familial and is associated with choreo-athetoid movements.

A 56-year-old male with complaints of low back pain is seen in your office. Symptoms started 3 months previously when he was lifting some boxes and felt a "pop." The pain is in the left lower lumbar area and extends into the upper left leg. On examination, the patient is tilting to the left side while sitting. His sensory exam of the lower extremity is inconsistent. His reflexes are muted. There is pain on left-leg raising that originates in the lumbar area and radiates to the left upper thigh. Which of the following evaluation techniques is most likely to demonstrate a functionally symptomatic disk herniation? A) Axial loading B) CT of lumbar region C) Electromyelography D) Lumbar MRI E) Lumbar spine series

Answer C. Electromyelography Explanation Back pain is a common complaint, and the severity ranges from simple muscle strains to denervating neuro-compression. Axial loading is a test of distraction that is not expected to reproduce intraspinal compressive lesions of the lumbar spine. Symptoms with axial loading are suggestive of a non-neurological etiology for the complaints. Plain radiography, as done in a lumbar series, can be helpful in cases of acute trauma to evaluate for fracture. CT scanning can be useful for detecting bony pathology but is less helpful for soft tissue abnormalities. MRI is the evaluation of choice for disk herniation. Unfortunately, these radiological modalities are limited to demonstrating structural changes and not functional compromise. Electromyelography is the only choice provided that is capable of demonstrating the function of the neuronal structures.

A 17-year-old woman is brought to the ED after having a witnessed generalized tonic-clonic seizure. She has a history of febrile seizures in childhood. On further questioning, you find that she has had episodic "staring" and unresponsiveness for the past two years. Observers report that she has lip smacking movements during the staring episodes. The episodes have been rare (about 4 per year), and she did not seek medical attention for these. She is otherwise well. Her exam is normal. What would be the least effective medication to treat these events? A) Phenobarbital B) Carbamazepine C) Ethosuximide D) Valproate E) Lamotrigine

Answer C. Ethosuximide Explanation The type of seizures that she has are complex partial—the staring associated with oral automatisms (the lip smacking)—and secondarily generalized tonic-clonic seizures. In other words, she has complex partial epilepsy. The febrile seizures are a risk factor for the later development of this type of epilepsy. For partial seizures, valproate, carbamazepine, phenobarbital, and lamotrigine would all be effective. Ethosuximide is effective only for absence seizures. Of the four possible agents, phenobarbital, carbamazepine, and valproate are teratogenic (pregnancy category D). Lamictal® is pregnancy category C, but may soon be switched to category B due to recent data that suggests that lamotrigine may be safe during pregnancy. This might be something to consider in a potentially sexually active teenager.

A 44-year-old man comes to the Emergency Department with quickly worsening weakness of both arms and legs. Four days ago, he had low back pain. Two days ago, he noticed weakness of both legs. Initially, he had difficulty only when climbing stairs, but now he has trouble walking. Yesterday, the weakness spread to both arms. He denies bowel or bladder dysfunction. He denies recent trauma. He had an upper respiratory infection about two weeks ago. There is no family history of neurologic disease. He denies risk factors for HIV. Physical Exam: He is awake, alert, and articulate. He has normal cranial nerves. He has 4/5 weakness in the upper and lower extremities bilaterally. The weakness is more pronounced in the distal extremities. Reflexes are 1+ in the arms. They are absent in the legs. Toes are downgoing bilaterally. There is no sensory level. There are no signs of cerebellar involvement. Which of the following is the most likely diagnosis? A) Chronic inflammatory demyelinating polyneuropathy (CIDP) B) Polyneuropathy C) Guillain-Barré syndrome D) Transverse myelitis E) Myasthenia gravis

Answer C. Guillain-Barré syndrome Explanation Based on his exam, he has a lower motor neuron disease (decreased reflexes, negative Babinski), so transverse myelitis is unlikely. Myasthenia causes weakness that worsens with exercise. CIDP is clinically similar to Guillain-Barré, except that it has a slower, more prolonged (>8 weeks) course. A polyneuropathy affects the sensory nerves, and causes a stocking-glove pattern of numbness. The two most common causes of polyneuropathy are diabetes and chronic alcohol abuse. Guillain-Barré can occur at any age. An upper respiratory infection, gastrointestinal infection, or immunization may precede the onset by 5 days to 3 weeks. Although an ascending symmetric weakness and loss of muscle stretch reflexes is the characteristic presentation, Guillain-Barré may cause cranial nerve weakness, oropharyngeal weakness, and numbness of the hands and feet. The cause is unknown but may be due to an autoimmune process. CSF analysis will show increased total protein and a normal white cell count. 20‒30% of patients will require mechanical ventilation. Consider mechanical ventilation for the 20, 30, 40 rule: VC < 20mL/kg, MIP less negative than - 30 cmH20 and MEP < 40 cmH20. Some autonomic dysfunction occurs in about 70% of patients.Plasmapheresis and IV immunoglobulin therapy (IVIG) are equally effective. Use one or the other if patients present within 4 weeks of initial symptoms. Do not use steroids because they are not effective. The usual course is for the disease to worsen for several days, plateau, and then begin resolving. Approximately 80% make a complete recovery and 10% have a prolonged illness.

A 14-year-old lethargic girl is brought to the ED. Her mom reports the daughter has had recent nausea, vomiting, and abdominal pain. In addition, she has been very thirsty and urinating very frequently.There is no recent fever, adenopathy, or malaise. On exam, she is quite sleepy, and has rapid, deep respirations. You think she might have a fruity smell to her breath. What is the most likely cause of her altered mental status? A) Hyperosmolar state B) Hypoglycemia C) Ketoacidosis D) Pancreatitis E) Hypercalcemia

Answer C. Ketoacidosis Explanation Diabetic ketoacidosis is the likely diagnosis. This occurs more often in juvenile diabetes and is due to insulin deficiency. The body cannot utilize serum glucose and breaks down fat as a fuel source, generating ketones. These can create a "fruity" smell to the breathas they are excreted through the oral mucosa. Physical symptoms may include polyuria and polydipsia, abdominal pain, nausea, and vomiting. Rapid, deep breathing may also occur. Nonketotic/hyperosmolar states may present with similar symptoms but occur much more often in adult onset diabetes (our patient is an adolescent), in a person who either forgets to take their insulin, eats sugary meals, or neglects to drink an appropriate amount of liquid. Hypoglycemia causes tremulousness, sweating, and hunger in association with lethargy. The treatment of ketoacidosis and hyperosmolar states is fluids, insulin, and repletion of electrolytes (sodium and potassium). Pancreatitis would be quite rare in this age group and hypercalcemia would not cause these symptoms.

An 80-year-old man presents to the emergency department following a seizure. He has a history of hypertension and chronic neck pain. MEDICATIONS: Amitriptyline, hydrochlorothiazide, tramadol, benazepril, and stool softeners His exam is unremarkable except for post-ictal confusion. LABORATORY: BUN 20, Cr 1.2, Na 140, K 4.2, Ca 9.8 Head CT with contrast: No abnormalities. Which of the following would you recommend next? A) Obtain lumbar puncture. B) Begin phenytoin. C) Stop tramadol. D) Obtain MRI. E) Stop amitriptyline.

Answer C. Stop tramadol. Explanation This older patient is evaluated for seizure. You are given the information that he has a negative contrast head CT scan. This makes tumor as a cause very unlikely. He is on tramadol, a medication that has been associated with seizures. This medication should be stopped, and the patient should have further evaluation only if he has additional seizures off this medication. His presentation is not suggestive of an infectious process such as meningitis or encephalitis, thus an LP is not indicated. AEDs should not be started after a single likely provoked tramadol seizure.

A 50-year-old man is brought to the emergency department by his wife. About 2 hours ago, he reported a sudden, severe headache. She went to get him Tylenol, and when she returned, he was lying on the couch and was difficult to arouse. He quickly awakened, but was confused. He seemed to drift off to sleep. When she returned a little later to check on him, he was unable to be aroused, at which point she called 911. There have been no recent illnesses. He is on no medications. Exam: His general exam is negative. He has a temperature of 100.2° F. He is unable to be aroused, but withdraws extremities to painful stimuli. Brainstem reflexes are intact. He has meningismus. There are no obvious focal neurological deficits, but your exam is limited. Which of the following is the most likely diagnosis? A) Herpes encephalitis B) Bacterial abscess C) Subarachnoid hemorrhage D) Large MCA infarct E) Intracranial tumor

Answer C. Subarachnoid hemorrhage Explanation The sequence of events is critical to the diagnosis. The headache was sudden: "Thunder-clap" and "Worst headache of my life" are common descriptors. It is not unusual for a person to lose consciousness with a subarachnoid bleed. Often, they appear better following this, and then worsen. The sign that shows meningeal irritation is meningismus. A low-grade fever is common. A stroke would not cause meningismus. Herpes is usually slower in onset, and associated with a high fever. Often, herpes causes changes in mentation prior to other symptoms. Abscesses and tumors are more likely to cause focal neurological deficits.

A 33-year-old woman sees her internist because of intermittent diplopia, and a feeling of overall fatigue for the last month. The double vision occurs mainly at night, usually while watching television. She denies associated symptoms such as dysphagia, dysarthria, weakness, or numbness. Prior to this, she was in excellent health. She has no other medical illness. There is a family history of hypertension. Her grandmother had a stroke at age 80. She is on prenatal multivitamins and is trying to become pregnant. She denies tobacco, alcohol, and drug use. PHYSICAL EXAMINATION: Vital signs are normal. General exam is negative. Neurologic exam reveals an alert, articulate woman. Her speech is without errors. Cranial nerve exam is remarkable for a mild left-sided ptosis. When asked to sustain upward gaze for 2 minutes, she complains of double vision. When she complains of the diplopia, a mild left 6th nerve weakness is evident. The remainder of her cranial nerves exam is normal. Motor, sensory, and coordination are normal. LABORATORY: Normal. Of the following tests, which would you perform first to confirm the diagnosis? A) EMG B) MRI of the brain and brainstem C) Tensilon® test (if available); if not available, then acetylcholine receptor antibodies (AChR-Ab) D) CT of the chest E) Anticholinesterase antibodies

Answer C. Tensilon® test (if available); if not available, then acetylcholine receptor antibodies (AChR-Ab) Explanation This patient has a characteristic presentation of myasthenia gravis. The age of onset is between the 2nd and 4th decades of life. Myasthenia is due to circulating polyclonal IgG autoantibodies (produced by B-lymphocytes) to the acetylcholine receptor. It is characterized by weakness that worsens with physical activity. In about 40% of cases, the presenting complaint is diplopia. Other common symptoms include dysarthria, dysphagia, and decreased range of facial movements. Limb and neck weakness is common. It is uncommon for the limbs to be affected without other symptoms. Medications such as D-penicillamine (once used to treat rheumatoid arthritis) can cause drug-induced myasthenia. If the drug is discontinued, the myasthenia-like syndrome resolves. Although TIA and stroke must be considered in the diagnosis, it is very unusual for ischemia to produce diplopia alone. Often, other neurological symptoms are present. Graves ophthalmopathy must always be considered when a patient has diplopia. Often, physical signs of Graves disease, such as the Graves stare, will suggest a diagnosis of thyroid disease. The first diagnostic test is the Tensilon test. When clinical signs are present (not just subjective diplopia), the Tensilon test can confirm the clinical diagnosis. Tensilon (edrophonium) is a short-acting acetylcholinesterase inhibitor that acts to prolong the effects of acetylcholine at the postsynaptic neuromuscular junction. This will reverse the weakness transiently. A positive test, then, would be improvement in clinical signs. A second test is the acetylcholine receptor antibody test. It is positive in 50% of patients with ocular myasthenia, and positive in 90% of patients with generalized myasthenia. In this case (ocular symptoms only), it would be a second-line test. However, the only manufacturer of edrophonium (Tensilon) stopped producing it in February 2008. So once supplies are exhausted, only the antibody test is available. CT of the chest would be indicated to eliminate the possibility of a thymoma, which is present in about 15% of patients with myasthenia gravis. EMG may be performed as a confirmatory test as well. The classic EMG finding in myasthenia gravis is the electro-decremental response to repetitive stimulation. An MRI of the brain would not be needed.

A 45-year-old woman presents with fever, irritability, headache, and meningismus. As part of the evaluation, a lumbar puncture is to be performed. Which of the following is true regarding a lumbar puncture if the needle's stylet is removed during insertion? A) To avoid infection, once removed, the stylet should not be replaced. B) The ease of locating the spinal space is increased. C) The risk of developing an epidermal tumor is increased. D) The risk of obtaining a "bloody tap" is decreased.

Answer C. The risk of developing an epidermal tumor is increased. Explanation Sampling of spinal fluid is critical in the management of meningitis. The sample is obtained by inserting a spinal needle with an inserted stylet into the spinal space. When the stylet is removed, spinal fluid flows through the needle and can be collected. Advancing the needle without a stylet in place increases the chance of displacing tissue. This tissue could block the needle, decreasing the chance of obtaining spinal fluid, or be displaced into the spinal space where it can form an epidermoid tumor. A bloody tap is most common when the needle advances too far, passes through the spinal space, and enters the vascular tissues on the far side. The entire process of lumbar puncture must be performed using sterile technique, and reinserting the stylet doesn't increase the risk of infection if sterile technique has been maintained.

A 34-year-old male presents to the clinic with complaints of unremitting and excruciating pain. It has been present for 2 years following a minor fall with bruising of the right thigh. He has been through several physicians and 2 pain clinics and states that he has been diagnosed with both reflex sympathetic dystrophy (RSD) and complex regional pain syndrome (CRPS). He is requesting extended release morphine, hydrocodone, and fentanyl patches. Which of the following findings is most consistent with the diagnosis of RSD? A) Abnormal reflexes in the affected limb B) Characteristic abnormal findings on EMG C) Dermatomal distribution of the pain D) Atrophy of dermal components in the affected area E) Elevation in the sedimentation rate

Answer D. Atrophy of dermal components in the affected area Explanation The term reflex sympathetic dystrophy has recently been replaced with the term complex regional pain syndrome type I. It arises as a severe pain following an often minor insult that is out of proportion to the known tissue damage. Patients with this process do demonstrate objective, but often subtle, physical findings during the course of the disease. These changes also vary over the clinical course. Vasomotor abnormalities, sudomotor abnormalities, or focal edema should be present to make the diagnosis. After the extended duration of disease noted in this scenario, the patient would be expected to have demonstrable atrophy to the skin structures of the affected area. There are no diagnostic EMG findings associated with CRPS. Though nerve involvement plays a role in the syndrome, it is not uncommon to have a non-dermatomal distribution. Reflexes are not affected by the process, and it is not inflammatory, so no elevation in the sedimentation rate should be expected.

An 83-year-old woman comes to the clinic with concerns about worsening dizziness. She has had an increase in disequilibrium recently, including a recent fall. She has no history of CAD or seizure disorder. Her symptoms begin when she stands up and starts to walk. They are improved when she stops for a minute and touches the wall. MEDICATIONS: Sertraline, nizatidine, CaCO3, and estrogen Which of the following is the most likely diagnosis? A) Vestibular neuronitis B) Orthostatic hypotension C) Benign positional vertigo D) Multiple sensory deficits E) Panic attacks

Answer D. Multiple sensory deficits Explanation This elderly patient notices disequilibrium when she walks. The key information is that her symptoms are improved when she touches a wall. She does not have vertigo symptoms. If she had orthostatic hypotension, her symptoms would not improve with touching a wall. Both benign positional vertigo and vestibular neuronitis have symptoms of vertigo, which this patient does not have. She is elderly, which is the population multiple sensory deficits are seen in. These patients may have visual, hearing, orthopedic, and neuropathy problems. The sum total of these sensory problems makes the patient feel unsteady. When sensory input from the hand (touching the wall) is included, she feels more stable.

A 68-year-old woman presents with a left arm tremor. She has had the tremor for about 3 years and recently was noticed to have developed a side-to-side head tremor as well. Her tremor worsens when she performs movements that involve her bringing her hand to her face, such as smoking or drinking coffee. Of note is that if she drinks beer, the tremor seems to improve. Which of the following is the likely etiology? A) Gilles de la Tourette syndrome B) Parkinson disease C) Myasthenia gravis D) Benign essential tremor E) Huntington disease

Answer D. Benign essential tremor Explanation Tremors can be catergorized as static and action tremors. Static tremors may occur at rest called resting tremors or may occur when the limbs are in a fixed posture called postural tremors. Action tremors remain constant during voluntary motion. Essential tremor (ET) casues postural and action tremors. The prevelance of essential tremor is 5% in the general population. Typically these patients do not have any neurological manifestations outside of the tremor. Tremor of the legs is very unusual in essential tremor. She has the classic symptoms of essential tremor. This occurs usually in the elderly and can be familial when it is called Familial tremor. The sporadic version is commonly refered to as benign essential tremor. Essential tremor improves with ingestion of alcohol. Treatment is with beta-blockers or primidone; the combination of both is more effective than monotherapy. Gabapentin and benzodiazepines are sometimes used as second or third line agents.The tremor associated with Parkinson disease has a characteristic appearance. The tremor is most obvious when the hand, arm, or leg is at rest, but it subsides with movement—unlike this patient's presentation. It is also important to remember that a hand tremor can take the form of a back-and-forth motion of the thumb and forefinger, as though rolling a small object between them. This is sometimes called "pill rolling." Usually "shuffling" gait or rigidity is described with Parkinson's as well. The other diseases listed will not present in this fashion.

A 47-year-old man is brought into the emergency department by his family because they thought they saw a seizure. When they first found him, he was lying on the couch, "shaking a little," and unresponsive. To the wife, he initially appeared "blue" around the lips. As he began to wake up, the family noticed right-sided weakness and "confusion" and called 911. On arrival to the emergency department, he is afebrile. There is no neck stiffness. The patient is confused, but is following simple commands. When he talks, there is great hesitancy. However, he can repeat short phrases. He has mild weakness of the right face and arm. There is weakness of the right leg as well, but much less noticeable than the face. He has no risk factors for heart disease (no hypertension, nonsmoker, no family history of heart disease). For several months, the family had noticed some word-finding problems, but they had joked that the patient was having "senile moments." He has been well, denied feeling sick, and has had no sick contacts. What is the most likely diagnosis? A) Herpes encephalitis B) Stroke C) Cerebral abscess D) Cerebral tumor

Answer D. Cerebral tumor Explanation The most common primary CNS tumor in adults aged 45-55 is glioblastoma multiforme or anaplastic astrocytoma. It most often arises in either the frontal or temporal lobe and can cause seizures. The presence of word difficulties before the observed seizure suggests that there is a cause for the seizure. The description of the word difficulties and the right-sided weakness place the lesion in the left frontal lobe. The absence of high fever or personality changes takes us away from a more serious infectious cause like abscess or encephalitis. He has no risk factors for heart disease, and is young, so stroke is unlikely.

A 56-year-old male presents for routine clinic visit with complaints of dizziness. He states the episodes are particularly common at night when he rolls over in bed. They last for 15-30 seconds and then resolve. The sensation is that of the room spinning around him. Appropriate diagnostic workup would include which of the following? A) MRI B) Audiogram C) Electronystagmography D) Dix-Hallpike maneuver E) Weber test

Answer D. Dix-Hallpike maneuver Explanation This patient has very brief episodes of vertigo, which usually occur when he has a sudden change of position such as rolling over in bed. This is the classic description of benign positional vertigo. This diagnosis is confirmed clinically with the Dix-Hallpike maneuver. You have the patient sit on the examination table with legs extended. You turn their head 30° to 45° toward one side and then help the patient quickly lie back so their head hangs over the end of the table. You then watch their eyes for involuntary eye movements (nystagmus). After they sit upright for a few minutes to recover from the vertigo, the procedure is repeated with their head turned in the opposite direction. There is no specific testing such as an audiogram ENG or MRI that would make this diagnosis. Additionally, he has no complaints of hearing loss.

Which of the following factors predicts a better prognosis in people who have multiple sclerosis? A) Many attacks within the first 2 years of the disease onset B) Early in the course of the illness, having high disability C) Male D) Few lesions on MRI E) Older age of onset

Answer D. Few lesions on MRI Explanation The list of factors that predict a poor prognosis are: 1) being male (women tend to do better), 2) onset of the illness at an older age, 3) having many relapses at the beginning of the illness (within the first 2 years) or starting out as having primary progressive MS, 4) having motor or cerebellar signs which do not significantly improve after an attack early on in the illness, 5) having high disability early in the course of the illness, and 6) having many lesions in the white matter on T2-weighted MRI imaging.

A 16-year-old quarterback is hit while playing in his high school football game. There is no loss of consciousness, but he can't remember what happened. He is able to walk off the field under his own power. He has never had a concussion before. You are the team physician, and the coach wants to know when his star quarterback can return to the game. His exam is normal except for the amnesia, so which of the following is your response to the coach? A) He can return to the game after 20 minutes if no symptoms other than the amnesia. B) Immediate hospitalization. C) He is out for 1 month at least. D) He is out of the current game but can return to play after 1 week if no symptoms at rest and exertion. E) He is out for the season.

Answer D. He is out of the current game but can return to play after 1 week if no symptoms at rest and exertion. Explanation A concussion results from trauma to the head. This can be due to a direct blow or hit to the head or indirect from a body blow as occurs in whiplash. Symptoms are due to neurometabolic dysfunction and include; dizziness, irritability, headaches, visual changes, disequilibrum and gait imbalance, amnesia, and transient loss of conciouness. There are 3 grades of concussion listed in the table below. Risks factors for concussions and predictors of a more protracted course include: younger age, prior concussion, history of migraines, high risk sports (football, hockey, lacrosse), and female gender. Concussion Grades Grade Confusion Amnesia Loss of Consciousness I Yes No No II Yes Yes No III Yes Yes Yes Concussion and Time Before Return to Contact Sports Grade Minimum Time to Return to Play Time Asymptomatic I 20 minutes At the initial exam II 1 week 1 week III 1 month 1 week He has a grade II concussion, which means no loss of consciousness but amnesia. With the first concussion of the season, the correct answer as described above is recommended. If it is the second concussion of the season, the athlete is out for a minimum of 1 month and then can return only if asymptomatic for 1 week. If this is the third concussion of the season, the athlete is out for the remainder of the season. A grade I concussion means confusion with no amnesia and no loss of consciousness. Grade III concussions are characterized by transient loss of consciousness or altered mental status at 24 hours. Evalaute at nearest ED (imaging etc) if LOC for over 1 minute, persistent symptoms and abnormal focal neurological signs, cervical injury with neck pain, C spine tenderness, or extremity motor or sensory defiects. For athletes who do not need an ED evaluation, consider imaging if symptoms or signs persist after 1 week.

A 31-year-old female presents with numbness in her left arm and some left leg weakness. She had an episode similar to this 9 months ago, which resolved spontaneously. Exam shows subjective difference in light touch and slight motor weakness in the left leg. Which of the following tests would you order? A) Head CT with contrast B) MRI of spinal cord C) CT without contrast D) Head MRI E) Nerve conduction studies of left arm

Answer D. Head MRI Explanation This 31-year-old woman has recurrent neurologic symptoms, with improvement between episodes. This history is suggestive of multiple sclerosis (MS). The most sensitive imaging procedure for MS is MRI. Although lesions can occur in the spinal cord, the brain is the best first imaging site. The brain MRI shows lesions in over 92% of patients with MS.

A 70-year-old comes to the office with his wife. He has been having problems walking for about 2 years, and starting about 6 months ago, has been having increasing difficulty with short-term memory. On further questioning, it seems that he has also had several episodes of urinary incontinence over the past 1 year. On exam, his gait has a "magnetic" quality. He scores 27/30 on mini-mental status exam, missing 2/3 items on recall, and not recalling today's date. His MRI shows enlarged ventricles, with transependymal flow. What is the most likely diagnosis? A) Parkinson disease B) Alzheimer's C) Creutzfeldt-Jakob Disease D) Normal pressure hydrocephalus

Answer D. Normal pressure hydrocephalus Explanation The magnetic gait and memory difficulties are characteristic of NPH. The third part of the "triad" for NPH is urinary incontinence. The enlarged ventricles support the diagnosis. Although gait problems occur in Parkinson's, the gait usually is slow, and has a festinating quality (not magnetic). Creutzfeldt-Jakob consists of a rapidly progressive dementia, with myoclonus, etc.

A 32-year-old man is brought in by his friends to the ED in a coma. The only available history is that he was found in the park by a jogger. His exam shows an obese, poorly developed man who is disheveled in appearance. He does not respond to noxious stimuli. He has bilateral pinpoint pupils. He does not smell of alcohol. There is no evidence of IV drug use. What is the most likely cause of the patient's coma? A) Nonketotic hyperosmolar coma B) Diabetic ketoacidosis C) Large middle cerebral artery infarct D) Overdose (opiates) E) Brainstem infarct

Answer D. Overdose (opiates) Explanation The bilateral pinpoint pupils tell us that it is either opiates or a pontine infarct. Although not impossible, a pontine infarct—as opposed to drug abuse—is very rare in a young person. A middle cerebral artery infarct does not cause coma and would manifest as focal neurological deficits. Diabetic coma is also unlikely because it would not cause pinpoint pupils. DKA would not present with these constellation of findings.

A 34-year-old man is being treated for his partial seizure disorder (he has simple partial and complex partial seizures due to a past head injury) with carbamazepine. Which of the following medications might decrease the serum concentration of carbamazepine? A) Valproate B) Lamotrigine C) Levetiracetam D) Phenobarbital E) Gabapentin

Answer D. Phenobarbital Explanation Carbamazepine is broken down by liver enzymes. Phenobarbital is a potent hepatic enzyme inducer, which will increase the metabolism of other medications which are hepatically metabolized. Valproate is a hepatic enzyme inhibitor and will increase serum carbamazepine levels. Lamotrigine is metabolized by glucuronic acid conjugation and will have no effect on carbamazepine. Levetiracetam and gabapentin are renally excreted and do not affect the metabolism of other antiseizure medications like carbamazepine.

A 48-year-old woman complains of "vision problems" which have gradually worsened over the past 6 months. She has no other complaints—no headaches, no weakness, numbness, etc. On exam, you find that she has diminished peripheral vision on both sides. You decide to order formal visual field testing to confirm this. What is your leading diagnosis? A) Cavernous sinus thrombosis B) Multiple sclerosis C) Anterior ischemic optic neuropathy D) Pituitary tumor

Answer D. Pituitary tumor Explanation Worldwide, the most likely cause of gradual peripheral vision loss is glaucoma (which is not on the list of choices). The bilateral peripheral visual field disturbances indicate a lesion of the optic nerve—more specifically the optic chiasm. Some people with pituitary tumors may also experience headaches, diplopia, and weight changes. The characteristic finding on formal visual field testing is a bitemporal hemianopsia. Although multiple sclerosis can cause vision problems, it would be unusual for MS to cause a bitemporal hemianopsia. Anterior ischemic optic neuropathy (AION) is a stroke in the eye—it causes visual disturbances in one eye. Most often, the person with AION notices the problem awakening. The patient notes seeing poorly in one eye: There is obscuration by a "dark shadow," often involving just the upper or lower half of vision, usually the area toward the nose. There is no pain in AION.

If you suspect HSV encephalitis in a patient, which test is most specific in confirming the diagnosis? A) MRI brain B) EEG C) CT brain D) Polymerase chain reaction (PCR) for HSV

Answer D. Polymerase chain reaction (PCR) for HSV Explanation A positive PCR for HSV confirms the diagnosis and is very specific. However, its sensitivity varies. The other tests can be helpful and are sometimes more sensitive.

A 58-year-old female presents complaining of a hand tremor that has been present for 2 years. Her family is concerned that she has Parkinson's. She notices the tremor daily. It rarely interferes with her functioning, but she is embarrassed in social situations. She notices that wine subdues the tremor, so she "always drinks at weekly sorority functions." Which of the following is the most appropriate intervention? A) Carbidopa/levodopa B) MRI of the posterior fossa C) Vascular evaluation of the posterior circulation D) Propranolol E) Chlordiazepoxide-weaning regimen for "gin fits" with follow-up in 4 days

Answer D. Propranolol Explanation You'll love this question when you see it on your Boards because you'll definitely know the answer. Recall that benign essential tremors improve with alcohol, and frequently the Boards give you this little hint. (But you must make sure they are not giving you a patient with alcohol withdrawal and a tremor!) This patient has no other symptoms of alcohol withdrawal, and she has a history of this tremor for 2 years, so this is not gin fits. This tremor is not the classic pill-rolling tremor of Parkinson's that occurs at rest and improves with movement. This tremor is present both at rest and with movement. MRI is not indicated to diagnose essential tremors. She has no symptoms of stroke, so vascular evaluation is incorrect. Benign essential tremors can be treated with propranolol or primidone.

A 70-year-old who has hypertension, hyperlipidemia, and diabetes, all of which is poorly controlled, comes to the emergency department with acute onset of left leg weakness. The patient's daughter first noticed this, and the patient seems to be relatively unconcerned about this. On exam, you find weakness of the arm as well, but less obvious than the leg. There is no fever, chills, or recent illness. Where is the most likely location of the stroke? A) Left anterior cerebral artery B) Right middle cerebral artery C) Left middle cerebral artery D) Right anterior cerebral artery

Answer D. Right anterior cerebral artery Explanation The symptoms are leg > arm, and on the left. Therefore, the stroke occurred in the right ACA territory. The patient's lack of concern suggests that he also has a neglect syndrome. This occurs more often in the setting of a nondominant (usually right-sided) parietal stroke, but can also occur with nondominant frontal infarcts. An MRI would help to confirm this.

A 26-year-old man is found unconscious and unresponsive in his well-ventilated apartment. No medical history is available. Fifteen minutes later, when he arrives in the emergency department, he has improved. PHYSICAL EXAMINATION: His general exam is negative. There is no evidence of trauma. His vital signs are normal. He is afebrile. There are no signs of meningeal irritation. He does not answer questions or follow commands well. Cranial nerves are normal. Formal strength testing is difficult, but he moves both arms and legs equally well. He withdraws all 4 limbs to painful stimuli. There is no decerebrate or decorticate posturing. Reflexes are 2+ and symmetric. Toes are downgoing bilaterally. LABS: Normal routine tests Which of the following is the best test to perform in the emergency department to narrow the differential? A) Skull x-ray B) CT scan of the brain C) Arterial blood gas D) Toxicology E) Lumbar puncture

Answer D. Toxicology Explanation Although several of the tests would probably be performed while the patient is in the emergency department, remember to look for a single, best answer. Given that there is no evidence of trauma, and no focal neurological deficits, the CT and skull x-ray are unlikely to help. The lack of meningeal signs eliminates diagnoses like encephalitis and subarachnoid hemorrhage, so the lumbar puncture is less likely to help. An ABG might help if carbon monoxide (CO) poisoning were suspected; however, we are told that his apartment was well ventilated. Also, CO poisoning occurs most often in older persons who use petroleum-burning space heaters in the winter. Although we see no evidence of drug intoxication on exam, toxicology would be very helpful in this situation.

A 55-year-old woman has horrible stabbing/shooting pains in the left cheek when she eats or brushes her teeth. There is no associated weakness, numbness, fever, chills, nausea, vomiting, or concurrent illness. The symptom has been present for about 2 weeks; it has not been getting worse. What is the most likely diagnosis? A) Tumor of the posterior fossa B) Multiple sclerosis C) Middle cerebral artery stroke D) Trigeminal neuralgia

Answer D. Trigeminal neuralgia Explanation Trigeminal neuralgia most often affects people > 30 years of age. It causes lancinating/stabbing pain. The pain occurs more often in V2 (cheek) > V3 (jaw) distributions of 5 th cranial nerve; it often has a trigger such as chewing, eating, or brushing teeth. Some of the most often used treatments are antiepileptic drugs like phenytoin, carbamazepine, oxcarbazepine, gabapentin, or pregabalin. If the person is a younger adult (< 40) or if there was other cranial nerve involvement, think of multiple sclerosis. (The demyelination can cause multiple cranial neuropathies all at the same time.) If several cranial nerves are involved and if the person has crossed findings (weakness on one side and numbness on the other), think of a tumor of the posterior fossa. Stroke would not be a likely cause of this symptom; in fact, strokes are very rarely painful. A stroke would not be recurrent.

A 26-year-old female attorney has a severe, retro-orbital, stabbing headache for twenty minutes. It awakens her from sleep each morning at 2 a.m. and has done so for the past 5 days. She has no prior history of headache. Medical history and family history are negative. She attended a wine-and-cheese festival in the Arts District downtown a week ago and purchased a box of Richart® chocolates from Paris. No allergies. No drugs or tobacco. Meds: oral contraceptive, MVI Which of the following is the most appropriate next step in patient care? A) Sumatriptan, single dose, and reassess in two hours B) Verapamil daily C) Ultrasound of the carotid arteries D) CT of the head with contrast E) 100% high-flow oxygen through mask and counsel to refrain from alcohol

Answer E. 100% high-flow oxygen through mask and counsel to refrain from alcohol Explanation This question describes a classic cluster headache pattern: Stabbing, retro-orbital headache Wakes from sleep Occurs daily for 5 days Now this question throws you a couple of added potential risk factors and asks you to interpret whether these facts are relevant to the case: oral contraceptives and recent increased intake of chocolate. Both are important risk factors for migraine headaches, but in the setting of a clear-cut presentation of a cluster pattern, these risk factors are not important. The Boards may give you cases of "red herring" risk factors, but they only do so when the case presentation is a clear-cut diagnosis. 100% high-flow oxygen promptly relieves a cluster headache. Alcohol also exacerbates cluster headaches, so it should be avoided. CT and ultrasound are useful only if her symptoms suggest a stroke (they don't). Sumatriptan and verapamil are used for migraine treatment and prophylaxis.

A 55-year-old woman has frequent sinus infections and has been taking azithromycin for the past 10 days for a recurrence of this. However, during the past week, she has had headaches and has developed right-sided proptosis and double vision. On exam, there is decreased movement of the right eye and diminished sensation over the right forehead and cheek (normal over the right chin). What is the cause of her neurological findings? A) Left middle cerebral artery stroke B) Ramsay-Hunt syndrome C) Right posterior communicating artery aneurysm D) Sagittal sinus thrombosis E) Cavernous sinus thrombosis

Answer E. Cavernous sinus thrombosis Explanation The presence of proptosis, ophthalmoplegia, and sensory disturbances in the V1 and V2 distribution of the 5 th cranial nerve all point to the right cavernous sinus as the location of the abnormality. In the cavernous sinus are V1, V2, III, IV, and VI. The sinusitis is a common cause of this (and needs to be treated with appropriate antibiotics). Sagittal sinus thrombosis might cause a venous infarct and would present with signs indicative of the region of the hemorrhage. Ramsay-Hunt is also known as herpes zoster oticus (aka zoster of the face). It presents with acute facial nerve paralysis, pain in the ear, taste loss in the front two-thirds of the tongue (facial nerve mediated), dry mouth and eyes, and eruption of a red vesicular rash in the ear canal, the tongue, or hard palate.

35-year-old woman presents with intense, sudden vertigo that causes her to feel very nauseous. She notices muffled hearing and ringing in the ears. Over the next few hours, the episode resolves. On further questioning, she describes several other episodes in her life, beginning at the age of 26. The attacks are not brought on by sudden head or body movements. There was no antecedent viral illness. She denies other neurological symptoms. What is the most likely diagnosis? A) Cerebellar infarct B) Acute vestibulitis C) Cerebellopontine angle tumor D) Benign positional vertigo E) Ménière disease

Answer E. Ménière disease Explanation Ménière's presents as the triad of vertigo, hearing loss, and tinnitus. The symptoms occur in unpredictable attacks. It usually begins in the 20s or 30s. It is rare in children. Sometimes, attacks are preceded by a sensation of fullness in the ear. As the disease progresses, the attacks, which last 1 to several hours at first, gradually increase in duration and intensity. Progressive hearing loss occurs, and persists in between the attacks. The treatments include restriction of sodium intake, and diuretics (such as furosemide or acetazolamide). Symptomatic treatment includes antiemetics and anti-vertigo agents (meclizine). Although cerebellar stroke can cause vertigo, it usually does not cause hearing loss. It would be very unusual for cerebellar stroke to cause recurrent episodes. Acute vestibulitis is usually post-viral and monophasic. It causes severe vertigo that typically lasts days to weeks. Benign positional vertigo (BPV) is more common in older adults and is precipitated by sudden head or body movements. The duration of vertigo in BPV is usually minutes and can be attenuated by repeating the action that causes the vertigo. Cerebellopontine angle tumors (usually schwannomas) cause progressive hearing loss, tinnitus, and vertigo. The symptoms do not relapse and remit.

A 20-year-old woman presents for evaluation of headaches. She has had headaches for the past 3 months, and they occur about 4 times a month. The headaches are extreme and involve the left side of her head. She reports maximum intensity behind her left eye. Her symptoms get worse with exercise. Usually, she will go into a quiet room with the lights off, and the headache will go away in 4 hours. On occasion, her headaches are associated with nausea. She has noted "flashing lights" before some of the headaches have occurred. A complete neurologic examination reveals no abnormalities. Which of the following do you recommend? A) CT scan with contrast B) CT scan without contrast C) MRI D) MRA E) No imaging at this time

Answer E. No imaging at this time Explanation She has migraines. She has had no change in her pattern, no seizures, and no focal neurologic findings or symptoms. It is recommended that no imaging be done in this patient.

A 36-year-old obese woman presents for evaluation of headaches. She has had increasing problems with headaches over the past 6 months. She has had no visual symptoms. She has occasional nausea but no focal neurologic symptoms. PHYSICAL EXAM: BP 120/70, P 80, skin—without lesions, fundi—normal examination, neurologic exam—without abnormalities MEDICATIONS: CaCO3, oral contraceptive pills, fluoxetine Which of the following is the most likely etiology for her headache? A) Prader-Willi syndrome B) Glioblastoma C) Pseudotumor cerebri D) Tuberous sclerosis E) Oral contraceptive pills

Answer E. Oral contraceptive pills Explanation This patient has had headaches for 6 months. The most likely diagnosis is headache associated with oral contraceptive pills. She has a small risk for pseudotumor cerebri because she is obese. This diagnosis is excluded by the normal funduscopic exam because papilledema is a hallmark of this. There are no cutaneous features suggestive of tuberous sclerosis (ash leaf spot, adenoma sebaceum).

A 65-year-old man comes to the office with his wife because, as she says, "He is moving so slow. I know we're not getting any younger, but he is really having troubles." Upon interviewing him, you discover that his voice has become softer, and he has had a tremor in the right hand. He has trouble turning over in bed, getting out of chairs without assistance, and has frequently "felt stuck" when trying to ambulate. His memory and thinking are normal. He is on no medications. Exam: His general exam is normal. Mental status exam is normal. His cranial nerves are remarkable for masked facies and hypophonic speech. He has increased tone (rigidity) on the right arm and leg, cogwheeling, and a resting tremor on the right. His gait is slow and festinating. He has a stooped posture. When walking, he turns "en bloc." Which of the following is the most likely diagnosis? A) Striatonigral degeneration B) Progressive supranuclear palsy (PSP) C) Shy-Drager D) Alzheimer's E) Parkinson disease

Answer E. Parkinson disease Explanation Since he is on no medications, he does not have a drug-induced Parkinsonism (not a choice here). Shy-Drager by definition includes severe orthostasis. Striatonigral degeneration looks like Parkinson's without the resting tremor. PSP includes loss of voluntary vertical gaze and axial rigidity (more than appendicular). PSP is also highly associated with dementia. Alzheimer's and Parkinson's can overlap. However, Alzheimer's does not begin with these symptoms, and is therefore very unlikely.

A 25-year-old woman presents with worsening weakness in her lower extremities. She denies headache, nausea, or vomiting. She has not had any significant illness except for a "stomach" flu 2 weeks ago. With this she had diarrhea for several days, but it has resolved. Physical Examination: HEENT: PERRLA, EOMI; No cranial nerve abnormalities Neck: Supple Heart: RRR without murmurs, rubs, or gallops Lungs: CTA Abdomen: + BS, soft, non-tender Extremities: No cyanosis, clubbing, or edema Neurological Exam: Marked weakness bilaterally over her lower extremities from the hip flexors down. She can barely lift her legs off the bed when in a lying position. Reflexes are absent in her lower extremities, and the brachioradialis reflex in her upper extremity is diminished bilaterally. Biceps tendon reflex appears to be intact. Lumbar puncture is done and shows only an elevated protein. Special studies are pending. The next day on examination her upper extremities are also becoming weak, and she has had some difficulty with breathing. Which of the following is the best therapy to begin at this point? A) Start interferon-beta therapy. B) Await ANA before proceeding with further therapy. C) 5 mg of prednisone. D) Start interferon-alpha therapy. E) Plasmapheresis.

Answer E. Plasmapheresis. Explanation This patient has Guillain-Barré syndrome. She has an ascending paralysis with loss of reflexes. Her CSF protein was elevated as an isolated finding. Plasmapheresis, if begun early, is very helpful. An additional clue here was the "stomach" virus episode—classically Guillain-Barré will follow Campylobacter infection or viral gastroenteritis. In acute inflammatory demyelinating polyneuropathy (Gullian-Barre Syndrome), 20‒30% of patients will require mechanical ventilation. Consider mechanical ventilation for the 20, 30, 40 rule: VC < 20mL/kg, MIP less negative than - 30 cmH20, MEP < 40 cmH20. Some autonomic dysfunction occurs in about 70% of patients.

A 26-year-old man is found unconscious and unresponsive in his well-ventilated apartment. No medical history is available. Fifteen minutes later, when he arrives in the emergency department, he has improved. PHYSICAL EXAMINATION: His general exam is negative. There is no evidence of trauma. His vital signs are normal. He is afebrile. There are no signs of meningeal irritation. He does not answer questions or follow commands well. Cranial nerves are normal. Formal strength testing is difficult, but he moves both arms and legs equally well. He withdraws all 4 limbs to painful stimuli. There is no decerebrate or decorticate posturing. Reflexes are 2+ and symmetric. Toes are downgoing bilaterally. LABS: Normal routine tests Which of the following is the most likely cause for his diminished mental state? A) Hyperglycemia (diabetes) B) Encephalitis C) Stroke D) Drug intoxication E) Seizure (post-ictal)

Answer E. Seizure (post-ictal) Explanation When confronted with a question like this, narrow the list first. Stroke is unlikely because of his age and the lack of focal neurologic findings. Encephalitis is unlikely without a fever and elevated white count. Hyperglycemia is out. The routine labs are negative. Drug intoxication is possible, as is a seizure/post-ictal state. Of these choices, seizure wins over drug intoxication because of the lack of findings on examination (no papillary abnormalities).

A 54-year-old male complains of episodes of dizziness (they last 25 minutes and resolve). He has been permanently dizzy since the last episode yesterday. Diplopia PMH: cigarettes and HTN Exam: regular heart rhythm and no carotid bruits Which of the following is most likely to result in the correct diagnosis? A) Empiric treatment with meclizine B) 3 sets of blood cultures C) Transesophageal echocardiogram D) Holter monitor E) Ultrasound of the posterior circulation in the neck

Answer E. Ultrasound of the posterior circulation in the neck Explanation The man in this case presents with dizziness. Immediately, you should think about the cerebellum and the posterior circulation. He appears to have episodic spells that improve in less than 24 hours. Then, he suffers "the big one" that fails to improve and brings him to medical attention. The case is describing TIAs of the posterior circulation, with an ultimate posterior circulation stroke that is associated with persistent dizziness and diplopia. The Board exam expects that you can differentiate the anterior and posterior carotid circulations based on symptoms. Don't forget: Anterior circulation goes to the middle cerebral artery. Those strokes look like aphasia and extremity weakness. Posterior circulation strokes have dizziness or vertigo as the main feature. While not included as a feature in this question, disruption of both the anterior and posterior carotid circulations can cause transient vision loss. Cervical carotid circulation TIAs/strokes are associated with monocular vision loss, while posterior circulation strokes are associated with bilateral homonymous vision loss (although patients often describe monocular vision loss). On the Boards, vision loss associated with diplopia or dizziness should be considered indicative of posterior circulation disease, and the patient should be evaluated with posterior carotid ultrasound. Transesophageal echocardiograms are not useful to evaluate the posterior circulation. Blood cultures are useful if you think this is a septic stroke from endocarditis, but nothing in the case suggests this diagnosis. Meclizine is sometimes prescribed for the acute symptoms of benign positional vertigo. A Holter monitor is used to diagnose arrhythmias—not indicated in this case because the diplopia and the dizziness clearly suggest posterior circulation disease.

A 22-year-old woman presents for evaluation of headaches. She has had headaches for the past 6 months, occurring 4-5 times a month. The headaches are of great intensity, involving the right side of her head with the maximum intensity of pain occurring behind her right eye. Symptoms worsen with exertion. Headaches last 3-6 hours, are sometimes associated with nausea, and on 2 occasions have been preceded by a scotoma in the right eye. Neurologic exam is unremarkable. Which of the following tests would you recommend next? A) CT scan without contrast B) MRI C) No imaging D) CT scan with contrast E) MRA

C. No imaging Explanation This patient has typical symptoms of migraine headaches. She is at a common age (22) for the onset of migraines. Migraines are more common in women. Aura occurs in about 25% of cases. The quality standards subcommittee of the American Academy of Neurology does not recommend the use of CNS imaging in adult patients with recurrent headaches typical of migraine, without focal neurologic signs or symptoms.

A 45-year-old man comes to his doctor's office with lower back pain. The pain started suddenly after he was lifting a couch as he and his wife were redecorating their living room. The pain radiates down the posterior right thigh to his right ankle. It is worse when he moves. On examination, he has difficulty performing strength testing because he is in pain (it hurts when he moves). The pain is brought out with right straight leg raises. He has a diminished right ankle jerk reflex. What is the most likely diagnosis? A) Right L3-4 radiculopathy B) Left L4-5 radiculopathy C) Right L4-5 radiculopathy D) Right L5-S1 radiculopathy

D. Right L5-S1 radiculopathy Explanation L4-5 and L5-S1 radiculopathies are the most common of the lumbar radiculopathies. L3-4 is the next most common. They occur most often in the 4 th and 5 th decades of life.

An 18-year-old woman complains of a left, unilateral, throbbing headache that was preceded by "flashing lights in the right eye." The headache began at a wine-and-cheese festival, worsened over 30 minutes, and then persisted for 3 hours. The patient now complains of right hand and foot weakness. Exam shows a mild right hemiparesis and word-finding difficulty. PMH is negative. Medications include an oral contraceptive pill and a daily multivitamin. No allergies. No drugs or tobacco. Which of the following is the most appropriate next step in patient care? A) 100% high-flow oxygen through mask and counsel to refrain from alcohol B) NSAID and close monitoring of response C) Verapamil daily D) CT of the head with contrast E) Ultrasound of the carotid arteries

Neurology Answer B. NSAID and close monitoring of response Explanation The good practicing internist can generate a 25-point differential diagnosis for the woman in this case, including rarities like secondary syphilitic meningitis and lupus encephalitis. But the internist who is going to pass the exam will zero in on this patient's presenting symptoms: Headache with preceding scotoma Wine-and-cheese festival Motor weakness Oral contraceptive pills The combination of the above spells migraine . The motor weakness is concerning, but in combination with the classic risk factors (aged cheese and OCPs) and the classic presentation (headache with scotoma), the most likely cause is a complicated migraine. High-flow oxygen is used to treat cluster headaches. Verapamil is useful for migraine prophylaxis and is not indicated to treat an acute migraine. Ultrasound of the carotids and CT scans are useful if you believe this woman has suffered a TIA/stroke, but her risks and her headache with scotoma are more consistent with migraine. Classic migraines can be treated with triptans, but remember that triptans and ergot drugs are to be avoided in complicated migraines because of the risk of stroke.


Ensembles d'études connexes

MDC 3 Rasmussen Module 1 Breast/Female Reproductive Disorders

View Set

State Laws & Regulations- GA P&C Exam Prep

View Set

Etchegoyen "La entrevista psicoanalítica: estructura y objetivos"

View Set

Unit 3 (Sem 4) Preeclampsia/HELLP/abruptio placentae

View Set